How to show that the Nearest Neighbour Algorithm given an instance of the metric travelling salesman problem...












1












$begingroup$


I would like to understand why the Nearest Neighbour Algorithm has a logarithmic approximation ratio when given an instance of the metric travelling salesman problem. I have come across the paper of Rosenkrantz et al. : https://disco.ethz.ch/courses/fs16/podc/readingAssignment/1.pdf



What I have difficulty in is Lemma 1 in this paper in the part where the lower bound $sum_{i in H} alpha_i l_i$ $geq$ 2 $sum_{i = k+1}^{min(2k, n)} alpha_i l_i$ is given. Can anyone help in clarifying where this lower bound has come from? Why is it assumed in the paper that the first $k$, $l_i$'s have $alpha_i$ = 0?










share|cite|improve this question











$endgroup$

















    1












    $begingroup$


    I would like to understand why the Nearest Neighbour Algorithm has a logarithmic approximation ratio when given an instance of the metric travelling salesman problem. I have come across the paper of Rosenkrantz et al. : https://disco.ethz.ch/courses/fs16/podc/readingAssignment/1.pdf



    What I have difficulty in is Lemma 1 in this paper in the part where the lower bound $sum_{i in H} alpha_i l_i$ $geq$ 2 $sum_{i = k+1}^{min(2k, n)} alpha_i l_i$ is given. Can anyone help in clarifying where this lower bound has come from? Why is it assumed in the paper that the first $k$, $l_i$'s have $alpha_i$ = 0?










    share|cite|improve this question











    $endgroup$















      1












      1








      1





      $begingroup$


      I would like to understand why the Nearest Neighbour Algorithm has a logarithmic approximation ratio when given an instance of the metric travelling salesman problem. I have come across the paper of Rosenkrantz et al. : https://disco.ethz.ch/courses/fs16/podc/readingAssignment/1.pdf



      What I have difficulty in is Lemma 1 in this paper in the part where the lower bound $sum_{i in H} alpha_i l_i$ $geq$ 2 $sum_{i = k+1}^{min(2k, n)} alpha_i l_i$ is given. Can anyone help in clarifying where this lower bound has come from? Why is it assumed in the paper that the first $k$, $l_i$'s have $alpha_i$ = 0?










      share|cite|improve this question











      $endgroup$




      I would like to understand why the Nearest Neighbour Algorithm has a logarithmic approximation ratio when given an instance of the metric travelling salesman problem. I have come across the paper of Rosenkrantz et al. : https://disco.ethz.ch/courses/fs16/podc/readingAssignment/1.pdf



      What I have difficulty in is Lemma 1 in this paper in the part where the lower bound $sum_{i in H} alpha_i l_i$ $geq$ 2 $sum_{i = k+1}^{min(2k, n)} alpha_i l_i$ is given. Can anyone help in clarifying where this lower bound has come from? Why is it assumed in the paper that the first $k$, $l_i$'s have $alpha_i$ = 0?







      algorithms computer-science






      share|cite|improve this question















      share|cite|improve this question













      share|cite|improve this question




      share|cite|improve this question








      edited Dec 10 '18 at 16:07







      Dylan Galea

















      asked Dec 10 '18 at 16:00









      Dylan GaleaDylan Galea

      103




      103






















          1 Answer
          1






          active

          oldest

          votes


















          0












          $begingroup$

          $k$ is at least half of the number of edges in $T$, otherwise $H$ would not be a complete subgraph on the node set. To obtain a lower bound now, we can just choose the $alpha_i$ suitable.



          $$sum alpha_i l_i = a_0 l_0 + ldots + a_kl_k + ldots + a_{min(2k,n)} l_{min(2k,n)}
          = 0 l_0 + ldots + 0l_k + ldots + 2 l_{k+1} + ldots + 2 l_{min(2k,n)}$$






          share|cite|improve this answer









          $endgroup$













          • $begingroup$
            but why are we assuming that $alpha_i = 2 $ $forall$ $i geq k+1$?
            $endgroup$
            – Dylan Galea
            Dec 13 '18 at 12:56












          • $begingroup$
            The $alpha_i$ have to sum to the number of edges in $T$ and can be at most 2. Therefore if we set all $alpha_0, ldots, alpha_k = 0$ we must set the rest of them to 2 so they add up to $2cdot (min(2k, n) - k -1)$
            $endgroup$
            – Loreno Heer
            Dec 13 '18 at 13:08












          • $begingroup$
            Though I think it must be shown that the lower bound holds for any configuration of the alphas. Though, since this holds other cases are trivial.
            $endgroup$
            – Dylan Galea
            Dec 13 '18 at 13:12










          • $begingroup$
            I think another important issue not mentioned in the paper is that the $l_i$'s must be positive.
            $endgroup$
            – Dylan Galea
            Dec 13 '18 at 18:00











          Your Answer





          StackExchange.ifUsing("editor", function () {
          return StackExchange.using("mathjaxEditing", function () {
          StackExchange.MarkdownEditor.creationCallbacks.add(function (editor, postfix) {
          StackExchange.mathjaxEditing.prepareWmdForMathJax(editor, postfix, [["$", "$"], ["\\(","\\)"]]);
          });
          });
          }, "mathjax-editing");

          StackExchange.ready(function() {
          var channelOptions = {
          tags: "".split(" "),
          id: "69"
          };
          initTagRenderer("".split(" "), "".split(" "), channelOptions);

          StackExchange.using("externalEditor", function() {
          // Have to fire editor after snippets, if snippets enabled
          if (StackExchange.settings.snippets.snippetsEnabled) {
          StackExchange.using("snippets", function() {
          createEditor();
          });
          }
          else {
          createEditor();
          }
          });

          function createEditor() {
          StackExchange.prepareEditor({
          heartbeatType: 'answer',
          autoActivateHeartbeat: false,
          convertImagesToLinks: true,
          noModals: true,
          showLowRepImageUploadWarning: true,
          reputationToPostImages: 10,
          bindNavPrevention: true,
          postfix: "",
          imageUploader: {
          brandingHtml: "Powered by u003ca class="icon-imgur-white" href="https://imgur.com/"u003eu003c/au003e",
          contentPolicyHtml: "User contributions licensed under u003ca href="https://creativecommons.org/licenses/by-sa/3.0/"u003ecc by-sa 3.0 with attribution requiredu003c/au003e u003ca href="https://stackoverflow.com/legal/content-policy"u003e(content policy)u003c/au003e",
          allowUrls: true
          },
          noCode: true, onDemand: true,
          discardSelector: ".discard-answer"
          ,immediatelyShowMarkdownHelp:true
          });


          }
          });














          draft saved

          draft discarded


















          StackExchange.ready(
          function () {
          StackExchange.openid.initPostLogin('.new-post-login', 'https%3a%2f%2fmath.stackexchange.com%2fquestions%2f3034099%2fhow-to-show-that-the-nearest-neighbour-algorithm-given-an-instance-of-the-metric%23new-answer', 'question_page');
          }
          );

          Post as a guest















          Required, but never shown

























          1 Answer
          1






          active

          oldest

          votes








          1 Answer
          1






          active

          oldest

          votes









          active

          oldest

          votes






          active

          oldest

          votes









          0












          $begingroup$

          $k$ is at least half of the number of edges in $T$, otherwise $H$ would not be a complete subgraph on the node set. To obtain a lower bound now, we can just choose the $alpha_i$ suitable.



          $$sum alpha_i l_i = a_0 l_0 + ldots + a_kl_k + ldots + a_{min(2k,n)} l_{min(2k,n)}
          = 0 l_0 + ldots + 0l_k + ldots + 2 l_{k+1} + ldots + 2 l_{min(2k,n)}$$






          share|cite|improve this answer









          $endgroup$













          • $begingroup$
            but why are we assuming that $alpha_i = 2 $ $forall$ $i geq k+1$?
            $endgroup$
            – Dylan Galea
            Dec 13 '18 at 12:56












          • $begingroup$
            The $alpha_i$ have to sum to the number of edges in $T$ and can be at most 2. Therefore if we set all $alpha_0, ldots, alpha_k = 0$ we must set the rest of them to 2 so they add up to $2cdot (min(2k, n) - k -1)$
            $endgroup$
            – Loreno Heer
            Dec 13 '18 at 13:08












          • $begingroup$
            Though I think it must be shown that the lower bound holds for any configuration of the alphas. Though, since this holds other cases are trivial.
            $endgroup$
            – Dylan Galea
            Dec 13 '18 at 13:12










          • $begingroup$
            I think another important issue not mentioned in the paper is that the $l_i$'s must be positive.
            $endgroup$
            – Dylan Galea
            Dec 13 '18 at 18:00
















          0












          $begingroup$

          $k$ is at least half of the number of edges in $T$, otherwise $H$ would not be a complete subgraph on the node set. To obtain a lower bound now, we can just choose the $alpha_i$ suitable.



          $$sum alpha_i l_i = a_0 l_0 + ldots + a_kl_k + ldots + a_{min(2k,n)} l_{min(2k,n)}
          = 0 l_0 + ldots + 0l_k + ldots + 2 l_{k+1} + ldots + 2 l_{min(2k,n)}$$






          share|cite|improve this answer









          $endgroup$













          • $begingroup$
            but why are we assuming that $alpha_i = 2 $ $forall$ $i geq k+1$?
            $endgroup$
            – Dylan Galea
            Dec 13 '18 at 12:56












          • $begingroup$
            The $alpha_i$ have to sum to the number of edges in $T$ and can be at most 2. Therefore if we set all $alpha_0, ldots, alpha_k = 0$ we must set the rest of them to 2 so they add up to $2cdot (min(2k, n) - k -1)$
            $endgroup$
            – Loreno Heer
            Dec 13 '18 at 13:08












          • $begingroup$
            Though I think it must be shown that the lower bound holds for any configuration of the alphas. Though, since this holds other cases are trivial.
            $endgroup$
            – Dylan Galea
            Dec 13 '18 at 13:12










          • $begingroup$
            I think another important issue not mentioned in the paper is that the $l_i$'s must be positive.
            $endgroup$
            – Dylan Galea
            Dec 13 '18 at 18:00














          0












          0








          0





          $begingroup$

          $k$ is at least half of the number of edges in $T$, otherwise $H$ would not be a complete subgraph on the node set. To obtain a lower bound now, we can just choose the $alpha_i$ suitable.



          $$sum alpha_i l_i = a_0 l_0 + ldots + a_kl_k + ldots + a_{min(2k,n)} l_{min(2k,n)}
          = 0 l_0 + ldots + 0l_k + ldots + 2 l_{k+1} + ldots + 2 l_{min(2k,n)}$$






          share|cite|improve this answer









          $endgroup$



          $k$ is at least half of the number of edges in $T$, otherwise $H$ would not be a complete subgraph on the node set. To obtain a lower bound now, we can just choose the $alpha_i$ suitable.



          $$sum alpha_i l_i = a_0 l_0 + ldots + a_kl_k + ldots + a_{min(2k,n)} l_{min(2k,n)}
          = 0 l_0 + ldots + 0l_k + ldots + 2 l_{k+1} + ldots + 2 l_{min(2k,n)}$$







          share|cite|improve this answer












          share|cite|improve this answer



          share|cite|improve this answer










          answered Dec 12 '18 at 23:31









          Loreno HeerLoreno Heer

          3,35411534




          3,35411534












          • $begingroup$
            but why are we assuming that $alpha_i = 2 $ $forall$ $i geq k+1$?
            $endgroup$
            – Dylan Galea
            Dec 13 '18 at 12:56












          • $begingroup$
            The $alpha_i$ have to sum to the number of edges in $T$ and can be at most 2. Therefore if we set all $alpha_0, ldots, alpha_k = 0$ we must set the rest of them to 2 so they add up to $2cdot (min(2k, n) - k -1)$
            $endgroup$
            – Loreno Heer
            Dec 13 '18 at 13:08












          • $begingroup$
            Though I think it must be shown that the lower bound holds for any configuration of the alphas. Though, since this holds other cases are trivial.
            $endgroup$
            – Dylan Galea
            Dec 13 '18 at 13:12










          • $begingroup$
            I think another important issue not mentioned in the paper is that the $l_i$'s must be positive.
            $endgroup$
            – Dylan Galea
            Dec 13 '18 at 18:00


















          • $begingroup$
            but why are we assuming that $alpha_i = 2 $ $forall$ $i geq k+1$?
            $endgroup$
            – Dylan Galea
            Dec 13 '18 at 12:56












          • $begingroup$
            The $alpha_i$ have to sum to the number of edges in $T$ and can be at most 2. Therefore if we set all $alpha_0, ldots, alpha_k = 0$ we must set the rest of them to 2 so they add up to $2cdot (min(2k, n) - k -1)$
            $endgroup$
            – Loreno Heer
            Dec 13 '18 at 13:08












          • $begingroup$
            Though I think it must be shown that the lower bound holds for any configuration of the alphas. Though, since this holds other cases are trivial.
            $endgroup$
            – Dylan Galea
            Dec 13 '18 at 13:12










          • $begingroup$
            I think another important issue not mentioned in the paper is that the $l_i$'s must be positive.
            $endgroup$
            – Dylan Galea
            Dec 13 '18 at 18:00
















          $begingroup$
          but why are we assuming that $alpha_i = 2 $ $forall$ $i geq k+1$?
          $endgroup$
          – Dylan Galea
          Dec 13 '18 at 12:56






          $begingroup$
          but why are we assuming that $alpha_i = 2 $ $forall$ $i geq k+1$?
          $endgroup$
          – Dylan Galea
          Dec 13 '18 at 12:56














          $begingroup$
          The $alpha_i$ have to sum to the number of edges in $T$ and can be at most 2. Therefore if we set all $alpha_0, ldots, alpha_k = 0$ we must set the rest of them to 2 so they add up to $2cdot (min(2k, n) - k -1)$
          $endgroup$
          – Loreno Heer
          Dec 13 '18 at 13:08






          $begingroup$
          The $alpha_i$ have to sum to the number of edges in $T$ and can be at most 2. Therefore if we set all $alpha_0, ldots, alpha_k = 0$ we must set the rest of them to 2 so they add up to $2cdot (min(2k, n) - k -1)$
          $endgroup$
          – Loreno Heer
          Dec 13 '18 at 13:08














          $begingroup$
          Though I think it must be shown that the lower bound holds for any configuration of the alphas. Though, since this holds other cases are trivial.
          $endgroup$
          – Dylan Galea
          Dec 13 '18 at 13:12




          $begingroup$
          Though I think it must be shown that the lower bound holds for any configuration of the alphas. Though, since this holds other cases are trivial.
          $endgroup$
          – Dylan Galea
          Dec 13 '18 at 13:12












          $begingroup$
          I think another important issue not mentioned in the paper is that the $l_i$'s must be positive.
          $endgroup$
          – Dylan Galea
          Dec 13 '18 at 18:00




          $begingroup$
          I think another important issue not mentioned in the paper is that the $l_i$'s must be positive.
          $endgroup$
          – Dylan Galea
          Dec 13 '18 at 18:00


















          draft saved

          draft discarded




















































          Thanks for contributing an answer to Mathematics Stack Exchange!


          • Please be sure to answer the question. Provide details and share your research!

          But avoid



          • Asking for help, clarification, or responding to other answers.

          • Making statements based on opinion; back them up with references or personal experience.


          Use MathJax to format equations. MathJax reference.


          To learn more, see our tips on writing great answers.




          draft saved


          draft discarded














          StackExchange.ready(
          function () {
          StackExchange.openid.initPostLogin('.new-post-login', 'https%3a%2f%2fmath.stackexchange.com%2fquestions%2f3034099%2fhow-to-show-that-the-nearest-neighbour-algorithm-given-an-instance-of-the-metric%23new-answer', 'question_page');
          }
          );

          Post as a guest















          Required, but never shown





















































          Required, but never shown














          Required, but never shown












          Required, but never shown







          Required, but never shown

































          Required, but never shown














          Required, but never shown












          Required, but never shown







          Required, but never shown







          Popular posts from this blog

          Biblatex bibliography style without URLs when DOI exists (in Overleaf with Zotero bibliography)

          ComboBox Display Member on multiple fields

          Is it possible to collect Nectar points via Trainline?